子机构
400-600-1123
logo
备考资讯 提分课程 答疑社区
登录
注册
题库解析   >   削弱 题型   >  
本题由chris提供

The proposal to hire ten new police officers in Middletown

is quite foolish. There is sufficient funding to pay the salaries of the new officers, but not the salaries of additional court and prison employees to process the increased caseload of arrests and convictions that new officers usually generate.

Which of the following, if true, will most seriously weaken the conclusion drawn above?

    A. Studies had shown that an increase in city’s police force does not necessarily reduce crime  

    B. When one major city increased its police force by 19% last year, there were 40% more arrests and 13% more convictions  

    C. If funding for the new police officers’ salaries is approved, support for other city services will have to be reduced during the next fiscal year  

    D. In most US cities, not all arrests result in convictions, and not all convictions result in prison terms 

    E. Middletown's ratio of police officers to citizens has reached a level at which an increase in the number of officers will have a deterrent effect on crime

登录申友雷哥GMAT,查看答案及解析

视频解析

暂无视频解析,点击获取更多视频内容

文字解析

答案:
E
引用cd上lawy
er的,觉得说的很好: 原文说增加警察会增加caseload of arrests and convictions that new officers usually generate。所以才得出结论增加警察是FOOLISH。E说的是增加警察下住了罪犯,使CASES没增加。所以WEAKEN结论。D说的是美国的大 体情况。原文在讲M的情况。没准M就是少数。所以无关。再说,即使D是说M的情况,也不对,原文是说增加ARRESTS就已经增加工作量了。而D只说 not all arrests result in convictions,没有说增加警察不一定增加ARRESTS。

GMAT会员

提交
OG视频
申友雷哥GMAT小助手

添加官方小助手微信
了解更多GMAT考试与咨询

100蜜糖购买当前课程

当前蜜糖数:颗 去获取

立即购买 取消购买

吉祥物小蜜蜂

关注公众号

公众号

扫码关注申友雷哥GMAT公众号

立即获取12GGMAT核心资料

微信咨询

申友在线咨询二维码图片

扫码添加申友雷哥GMAT官方助手

立即咨询GMAT网课面授课程

联系申友雷哥 全国免费咨询热线:400-600-1123

Copyright © 2021 All Right Reserved 申友雷哥教育 版权所有 沪ICP备17005516号-3 免责声明 互联网经营许可证编号:沪B2-20210282